문제

다음 식을 만족하는 non-constant 다항식 \(P(x), Q(x)\)가 존재하는지 판별하시오.


$$ P(x)^{10}+P(x)^9 = Q(x)^{21}+Q(x)^{20} $$


스포 방지선

________________________________________________________________________________________________________________________
















________________________________________________________________________________________________________________________


이 문제도 얘들이 많이 못 풀었는데, 내가 시험장에서 깔끔하게 푼 문제다. 여기서도 점수 좀 딴 듯 ㅋㅋ


(증명) 불가능하다. 귀류법으로 보이자. 조건을 만족하는 non-constant 다항식 \(P, Q\)가 존재한다 가정하자.

다항식 \(f\)에 대하여 \(S_f\)를 \(f(x) = 0\)의 (복소수) 해집합이라 하자. 중근은 한 번만 센다. 다음 관찰을 하자.


관찰 1. \(S_{P} \cap S_{P+1} = S_{Q} \cap S_{Q+1} = \emptyset\)이고, \(S_P \cup S_{P+1} = S_{Q} \cup S_{Q+1} \)이다.

관찰 2. 주어진 식 양변의 차수를 비교하면, \(\text{deg}(P)=21x , \text{deg}(Q)=10x\)인 자연수 \(x\)가 있음을 알 수 있다.  

관찰 3. \(|S_{f}| \le \text{deg} (f)\)가 성립한다. 그러므로 \( |S_P| + |S_{P+1}| = |S_Q|+|S_{Q+1}| \le 20x \)다.


이제 여기서 다음 Lemma를 보이자. 


Lemma. non-constant 다항식 \(f\)에 대하여, \(|S_f| + |S_{f+1}| \ge \text{deg}(f) +1 \)가 성립한다. 


Proof of Lemma. 우선 \( \displaystyle f(x) = \prod_{i=1}^n (x-r_i)^{c_i} \), \( \displaystyle f(x)+1 = \prod_{i=1}^m (x-r'_i)^{c'_i}\)라 하자.

여기서 \(n=|S_f|\), \(m=|S_{f+1}|\)이다. \(r_i, r'_i\)들은 앞선 관찰 1에 의해 모두 서로 다른 복소수다.

한편, \(f'(x)\)는 \(r_i\)를 \(c_i-1\)-중근으로 가지고, \(r'_i\)를 \(c'_i-1\)-중근으로 가진다. 그러므로

$$ \text{deg}(f)-1 =\text{deg}(f')  \ge \sum_{i=1}^n (c_i-1) + \sum_{i=1}^m (c'_i-1) = 2 \text{deg}(f)-n-m$$

가 성립하고, 정리하면 \( |S_f|+ |S_{f+1}| \ge \text{deg}(f)+1\)을 얻는다. \(\blacksquare\)


Lemma와 관찰 3으로 \(20x \ge |S_Q|+|S_{Q+1}| = |S_P|+|S_{P+1}| \ge \text{deg}(P)+1 = 21x+1\)을 얻고, 이는 모순. \(\blacksquare\)